Proofs in analytic geometry and vector spaces.

Click For Summary
The discussion centers on the validity of using vector coordinates to prove geometric statements, specifically the orthogonality of a square's diagonals. The proposed proof involves calculating the dot product of the diagonals in a three-dimensional space, R3, while assuming an orthogonal basis. Concerns are raised about the generality of the proof, particularly regarding the assumptions made about the vector space and basis. The conversation suggests that a proof using plane geometry might be more universally applicable. Ultimately, the need for a more general proof is emphasized, particularly in relation to the choice of vector space.
LCSphysicist
Messages
644
Reaction score
162
Homework Statement
All below
Relevant Equations
N/A
I was just thinking, if is said to me demonstrate any geometry statement, can i open the vector in its vector's coordinates? I will say more about:

For example, if is said to me: Proof the square's diagonals are orthogonal, how plausible is a proof like?:

d1 = Diagonal one = (a,b,c)
d2 = Diagonal two =(-a,-b,c)

but a² + b² = l² = c²

d1*d2 = (-a² -b² + c²) = (-(a² + b²) + c²) = > This is the dot product

The problems i see is:

#1 = I adopt a R3 space.
#2 = I assumed the basis orthogonal, so the distance between the vertex lying in the same side is (a²+b²)^(1/2)

The dot product holds for any basis anyway.

Can someone say: "(This is not a general proof, you adopt the orthogonal basis and R3)"?

We could try to proof by plane geometry, this is the general proof?

OBS: There is too
1594786685858.png

1594786698232.png
(the rest is easy)

What seems to me more general proof.
 
Physics news on Phys.org
There is indeed a problem with #2, you have to prove that it holds in any base but i don't understand the problem with #1, how are you supposed to work with vectors if you don't adapt a vector space(R^3 or R^2)?

You are right that the proof with u and v is more general more complete i would say.
 
Question: A clock's minute hand has length 4 and its hour hand has length 3. What is the distance between the tips at the moment when it is increasing most rapidly?(Putnam Exam Question) Answer: Making assumption that both the hands moves at constant angular velocities, the answer is ## \sqrt{7} .## But don't you think this assumption is somewhat doubtful and wrong?

Similar threads

  • · Replies 9 ·
Replies
9
Views
3K
  • · Replies 4 ·
Replies
4
Views
2K
Replies
1
Views
1K
  • · Replies 16 ·
Replies
16
Views
3K
  • · Replies 2 ·
Replies
2
Views
2K
  • · Replies 15 ·
Replies
15
Views
3K
Replies
5
Views
2K
  • · Replies 1 ·
Replies
1
Views
2K
  • · Replies 9 ·
Replies
9
Views
4K
  • · Replies 18 ·
Replies
18
Views
2K